Residue at a pole of non-integer order

  • Thread starter Thread starter emob2p
  • Start date Start date
  • Tags Tags
    Pole Residue
Click For Summary
Calculating the residue at a pole of non-integer order presents challenges due to the function's non-analytic nature. The function in question, e^(ipx)/(p - iκ)^(η), where η is a positive non-integer, raises concerns about defining poles and residues for non-meromorphic functions. Meromorphic functions are typically analytic except at discrete points, but the presence of non-integer powers complicates this, as they involve multiple branches and require branch cuts. The residue at the pole p = iκ may indeed be zero, but a formal proof is needed to confirm this. The discussion highlights the complexities of integrating through branch cuts when dealing with non-integer powers.
emob2p
Messages
56
Reaction score
1
Hi,
Does anyone know a straightforward way to calculate a residue at at a pole of non-integer order. I'm trying to find the residue of \frac {e^{ipx}}{(p - i \kappa)^\eta} at p = i \kappa where \eta is a positive non-integer. Thanks.

I have reason to suspect it's zero, but I'd need to see the proof.
 
Last edited:
Physics news on Phys.org
How are you defining poles and residues for non-meromorphic functions?
 
I'm not sure what non-meromorphic functions are.
 
A meromorphic function is simply a function that is almost analytic in its domain. In detail, it is analytic except at a discrete set of points, and at these points it cannot have an essential singularity. Meromorphic functions are what you usually meet in the theory of residues partly because these functions are "nice" almost everywhere so many of the theorems concerning analytic functions apply. The trouble with your function is that it isn't analytic because it has a non integer power. Non integer powers are defined using the logarithm and therefore necessarily have multiple branches and require a branch cut. In this case, the cut has to start at p = i \kappa so you have to integrate through the branch cut no matter how you define it.
 
Last edited:
Monkey, you're everywhere. Thanks again, it makes good sense.
 
Question: A clock's minute hand has length 4 and its hour hand has length 3. What is the distance between the tips at the moment when it is increasing most rapidly?(Putnam Exam Question) Answer: Making assumption that both the hands moves at constant angular velocities, the answer is ## \sqrt{7} .## But don't you think this assumption is somewhat doubtful and wrong?

Similar threads

  • · Replies 14 ·
Replies
14
Views
3K
Replies
1
Views
1K
Replies
3
Views
3K
Replies
4
Views
2K
Replies
6
Views
2K
  • · Replies 12 ·
Replies
12
Views
3K
  • · Replies 2 ·
Replies
2
Views
3K
  • · Replies 4 ·
Replies
4
Views
1K
  • · Replies 1 ·
Replies
1
Views
2K
  • · Replies 2 ·
Replies
2
Views
5K